Rudin Solved
Rudin Solved
mo
~8
1976
)upp.
\/lATH
Chapter 1
Exercise 1.2 Prove that there is no rational number whose square is 12.
First Solution. Since v'f2 = 2.)3, we can inv~ke the previous problem and
prove that .J3 is irrational. If m and n are integers having no common factor
and such that m 2 ....:. 3n 2 , then m is divisible by 3 (since if m 2 is divisible by 3,
so ism). Let m = 3k. Then m 2 = 9k 2 , and we have 3k 2 = n 2 . It then follows
that n is also divisible by 3 contradicting the assumption that m and n have no
common factor.
Exercise 1.3 Prove Proposition 1.15, i.e., prove the following statements:
(a) If x =f. 0 and xy = xz, then y = z.
(b) If x =f. 0 and xy = x, then y = 1.
(c) If x =f. 0 and xy = 1, then y = 1/x.
(d) If x =f. 0, then 1/(1/x) = x.
Solution. (a) Suppose x ::J. 0 and xy = xz. By Axiom (M5) there exists an
element 1/x such that 1/x = 1. By (M3) and (M4) we have (1/x)(xy) =
((1/x)x)y = ly = y, and similarly (1/x)(xz) = z. Hence y = z.
(b) Apply (a) with z = 1.
(c) Apply (a) with z = ljx.
(d) Apply (a) with x replaced by 1/x, y = 1/(1/x), and z = x.
Exercise 1.5 Let A be a nonempty set of real numbers which is bounded below.
Let -A be the set of all numbers -x, where x EA. Prove that
Solution: We need to prove that -sup( -A) is the greatest lower bound of A.
For brevity, let a= -sup( -A). We need to show that a:::; x for all x E A and
a ~ f3 if f3 is any lower bound of A.
Suppose x EA. Then, -x E -A, and, hence -x:::; sup( -A). It follows that
x ~ -sup( -A), i.e., a:::; x. Thus a is a lower bound of A.
Now let f3 be any lower bound of A. This means f3 :::; x for all x in A.
Hence -x:::; -{3 for all x E A, which says y:::; -{3 for ally E -A. This means
-{3 is an upper bound of -A. Hence -{3 ~ sup( -A) by definition of sup, i.e.,
f3:::; -sup( -A), and so- sup( -A) is the greatest lower bound of A.
(c) If x is real, define B (x) to be the set of all numbers bt, where t is rational
and t ::; x. Prove that
br = supB(r)
when r is rational. Hence it makes sense to define
bx =sup B(x)
c = (cjm)m < uv E B(x + y), and soc is not an upper bound for B(x + y). It
follows that supB(x) supB(y) is the least upper bound of B(x + y), i.e.,
as required.
Exercise 1.7 Fix b > 1, y > 0, and prove that there is a unique real x such
that bx = y, by completing the following outline. (This xis called the logarithm
of y to the base b.)
(a) For any positive integer n, bn - 1 2: n(b - 1).
(b) Hence b-12: n(b1 fn -1).
(c) If t > 1 and n > (b- 1)/(t- 1), then b1 fn < t.
(d) If w is such that bw < y, then bw+(l/n) < y for sufficiently large n; to see
this apply part (c) with t = y · b-w.
(e) If bw > y, then bw-(l/n) > y for sufficiently large n.
(f) Let A be the set of all w such that bw < y, and show that x = sup A
satisfies bw = y.
(g) Prove that this x is unique.
Solution. (a) The inequality bn - 1 2: n(b- 1) is equality if n = 1. Then, by
induction bn+l -1 = bn+ 1 -b+ (b-1) = b(bn -1) + (b-1) 2: bn(b-1) + (b-1) =
(bn + 1)(b- 1) 2: (n + 1)(b- 1).
(b) Replace b by b1 fn in part (a).
(c) The inequality n > (b -1)/(t- 1) can be rewritten as n(t- 1) > (b -1),
and since b- 1 2: n(b 1 fn- 1), we have n(t- 1) > n(b 1 fn- 1), which implies
t > blfn.
(d) The application of part (c) with t = y · b-w > 1 is immediate.
(e) The application of part (c) with t = bw · (1 j y) yields the result, as in
part (d) above.
(f) There are only three possibilities for the number x =sup A: 1) bx < y; 2)
bx > y; 3) bx = y. The first assumption, by part (d), implies that x+ (1/n) E A
for large n, contradicting the assumption that xis an upper bound for A. The
second, by part (e), implies that x- (1/n) is an upper bound for A if n is large,
contradicting the assumption that x is the smallest upper bound. Hence the
only remaining possibility is that bx = y.
(g) Suppose z =/:. x, say z > x. Then bz = bx+(z-x) = bxbz-x > bx = y.
Hence x is unique. (It is easy to see that bw > 1 if w > 0, since there is a
positive rational number r = 7: with 0 < r < w, and br = (bm )1 /n. Then
bm > 1 since b > 1, and (bm)lfn > 1 since 1n = 1 < bm.)
9
Exercise 1.8 Prove that no order can be defined in the complex field that turns
it into an ordered field. Hint: -1 is a square.
Solution. By Part (a) of Proposition 1.18, either i or -i must be positive. Hence
-1 = i 2 = ( -i) 2 must be positive. But then 1 = ( -1) 2, must also be positive,
and this contradicts Part (a) of Proposition 1.18, since 1 and -1 cannot both
be positive.
Now
a2 _ b2 = !w! + u _ !w! - u = u
2 2 '
and, since (xy) 112 . x 112y 112,
2
_ (!w!+u!w!-u)l/2_ (!w! -u2)1/2
2ab - 2 - 2 .
2 2 4
10 CHAPTER 1. THE REAL AND COMPLEX NUMBER SYSTEMS
Hence
2ab = 2( (~)
2
r
12
Exercise 1.11 If z is a complex number, prove that there exists an r > 0 and a
complex number w with lwl = 1 such that z = rw. Are wand r always uniquely
determined by z?
Solution. If z =' 0, we take r = 0, w = 1. (In this case w is not unique.)
Otherwise we taker= !zl and w = z/lzl, and these choices are unique, since if
z = rw, we must haver= r!wl = !rw! = !z!, zjr.
Solution. The case n = 2 is Part (e) of Theorem 1.33. We can then apply this
result and induction on n to get
Exercise 1.14 If z is a complex number such that \z\ = 1, that is, such that
zz = 1, compute
\1 + z\ 2 + \1- z\ 2 = 4.
Exercise 1.15 Under what conditions does equality hold in the Schwarz in-
equality?
Solution. The proof of Theorem 1.35 shows that equality can hold if B = 0 or
if Baj- Cb.i = 0 for all j, i.e., the numbers aj are proportional to the numbers
bj. (In terms of linear algebra this means the vectors a = (a1, a2, ... , an) and
b = (bll b2, ... , bn) in complex n-dimensional space are linearly dependent. Con-
versely, if these vectors are linearly independent, then strict inequality holds.)
\z - x\ = \z - y\ = r.
(b) If 2r = d, there is exactly one such z.
(c) If 2r < d, there is no such z.
How must these statements be modified if k is 2 or 1?
Solution. (a) Let w be any vector satisfying the following two equations:
w·(x-y)
\w\2
From linear algebra it is known that all but one of the components of a solution
w of the first equation can be arbitrary. The remaining component is then
uniquely determined. Also, if w i~ any non-zero solution of the first equation,
there is a unique positive number t such that tw satisfies both equations. (For
example, if x 1 =/:. y1 , the first equation is satisfied whenever
Zl =
Z2(X2- Y2) + ... + Zk(Xk - Yk)
.
Yl- Xl
If (z 1 , z2, ... , zk) satisfies this equation, so does (tz 1 , tz2, ... , tzk) for any real
number t.) Since at least two of these components can vary independently, we
can find a solution with these components having any prescribed ratio. This
12 CHAPTER 1. THE REAL AND COMPLEX NUMBER SYSTEMS
ratio does not change when we multiply by the positive number t to obtain
a solution of both equations. Since there are infinitely many ratios, there are
infinitely many distinct solutions. For each such solution w the vector z -
~x + ~y + w is a solution of the required equation. For
ly;x+wl2
y-xl2 x-y 2
l -2- +2w·-2-+lwl
·d2 d2
- +0+r 2 - -
4 4
r2
'
and a similar relation holds for iz - y! 2 .
(b) The proof of the triangle inequality shows that equality can hold in this
inequality only if it holds in the Schwarz inequality, i.e., one of the two vectors
is a scalar multiple of the other. Further examination of the proof shows that
the scalar must be nonnegative. Now the conditions of this part of the problem
show that
!x- Yl = d = !x- zi + iz - Yl·
Hence it follows that there is a nonnegative scalar t such that
x- z = t(z- y).
However, the hypothesis also shows immediately that t = 1, and so z is uniquely
determined as
x+y
Z= -2-.
(c) If z were to satisfy this condition, the triangle inequality would be vio-
lated, i.e., we would have
Exercise 1.18 If k 2:: 2 and x E Rk, prove that there exists y E Rk such that
y =I= 0 but x · y = 0. Is this also true if k = 1?
Solution. If x has any components equal to 0, then y can be taken to have
the corresponding components. equal to 1 and all others equal to 0. If all the
components of x are nonzero, y can be taken as ( -x 2 , x 1 , 0, ... , 0). This is, of
course, not true when k = 1, since the product of two nonzero real numbers is
nonzero.
Exercise 1.19 Suppose a E Rk, bE Rk. Find c E Rk and r > 0 such that
lx - ai = 2jx- hi
lx-al = 2lx- bl
4 1 I -lb-al
2
Ix- -b+
3 -a=
3 3 .
If we square both sides of both equations, we an equivalent pair of equations,
the first of which reduces to
and the second of which reduces to this equation divided by 3. Hence these
equations are indeed equivalent.
Exercise 1.20 With reference to the Appendix, suppose that property (III)
were omitted from the definition of a cut. Keep the same definitions of order
and addition. Show that the resulting ordered set has the least-upper-bound
property, that addition satisfies axioms (A1) to (A4) (with a slightly different
zero element!) but that (A5) fails.
Solution. We are now defining a cut to be a proper subset of the rational
numbers that contains, along with each of its elements, all smaller rational
14 CHAPTER 1. THE REAL AND COMPLEX NUMBER SYSTEMS